Seeking Methods to solve $Fleft(alpharight) = int_{0}^{1} x^alpha arcsin(x):dx$












5












$begingroup$


I'm looking for different methods to solve the following integral.
$$ Fleft(alpharight) = int_{0}^{1} x^alpha arcsin(x):dx$$



For $alpha > 0$



Here the method I took was to employ integration by parts and then call to special functions, but can this equally be achieved with say a Feynman Trick? or another form integral transform?



My approach in detail:



Employ integration by parts:



begin{align}
v'(x) &= x^alpha & u(x) &= arcsin(x) \
v(x) &= frac{x^{alpha + 1}}{alpha + 1} & u'(x) &= frac{1}{sqrt{1 - x^2}}
end{align}



Thus,



begin{align}
Fleft(alpharight) &= left[frac{x^{alpha + 1}}{alpha + 1}cdotarcsin(x)right]_0^1 - int_0^1 frac{x^{alpha + 1}}{alpha + 1} cdot frac{1}{sqrt{1 - x^2}} :dx \
&= frac{pi}{2left(alpha + 1right)} - frac{1}{alpha + 1}int_0^1 x^{alpha + 1}left(1 - x^2right)^{-frac{1}{2}} :dx
end{align}



Here make the substitution $u = x^2$ to obtain



begin{align}
Fleft(alpharight) &= frac{pi}{2left(alpha + 1right)} - frac{1}{alpha + 1}int_0^1 left(sqrt{u}right)^{alpha + 1}left(1 - uright)^{-frac{1}{2}} frac{:du}{2sqrt{u}} \
&= frac{pi}{2left(alpha + 1right)} - frac{1}{2left(alpha + 1right)}int_0^1 u^{frac{alpha}{2}}left(1 - uright) ^{-frac{1}{2}} :du \
&= frac{1}{2left(alpha + 1right)} left[ pi - Bleft(frac{alpha + 2}{2}, frac{1}{2} right) right]
end{align}



begin{align}
Fleft(alpharight) &=frac{1}{2left(alpha + 1right)} left[ pi - frac{Gammaleft(frac{alpha + 2}{2}right)Gammaleft(frac{1}{2}right)}{Gammaleft(frac{alpha + 2}{2} + frac{1}{2}right)} right] \
&= frac{1}{2left(alpha + 1right)} left[ pi - frac{Gammaleft(frac{alpha + 2}{2}right)sqrt{pi}}{Gammaleft(frac{alpha + 3}{2}right) } right] \
&= frac{sqrt{pi}}{2left(alpha + 1right)} left[ sqrt{pi} - frac{Gammaleft(frac{alpha + 2}{2}right)}{Gammaleft(frac{alpha + 3}{2}right) } right]
end{align}



Edits:
Correction of original limit observation (now removed)
Correction of not stating region of convergence for $alpha$.
Correction of 1/sqrt to sqrt in final line.



Thanks to those commentators for pointing out.










share|cite|improve this question











$endgroup$








  • 1




    $begingroup$
    How would you expect to get $Gamma$'s out of Feynman Trick or whatever else? (And, BTW, $displaystylelim_{alphatoinfty}F(alpha)=0$).
    $endgroup$
    – metamorphy
    Dec 10 '18 at 1:03






  • 2




    $begingroup$
    There needs to be some restrictions on the value of $alpha$ otherwise the improper integral will not converge, namely $alpha > -2$. The case $alpha = -1$ also needs to be handled with care.
    $endgroup$
    – omegadot
    Dec 10 '18 at 2:12






  • 1




    $begingroup$
    Yes, sorry. I meant to say it only converges for $alpha > -2$ and diverges for $alpha leqslant -2$.
    $endgroup$
    – omegadot
    Dec 10 '18 at 3:02






  • 1




    $begingroup$
    I'm fairly certain that's the fastest way to do it
    $endgroup$
    – clathratus
    Dec 10 '18 at 3:29






  • 1




    $begingroup$
    The $frac{1}{sqrt{pi }}$ in the last line needs to be $sqrt{pi}$.
    $endgroup$
    – JimB
    Dec 10 '18 at 5:12
















5












$begingroup$


I'm looking for different methods to solve the following integral.
$$ Fleft(alpharight) = int_{0}^{1} x^alpha arcsin(x):dx$$



For $alpha > 0$



Here the method I took was to employ integration by parts and then call to special functions, but can this equally be achieved with say a Feynman Trick? or another form integral transform?



My approach in detail:



Employ integration by parts:



begin{align}
v'(x) &= x^alpha & u(x) &= arcsin(x) \
v(x) &= frac{x^{alpha + 1}}{alpha + 1} & u'(x) &= frac{1}{sqrt{1 - x^2}}
end{align}



Thus,



begin{align}
Fleft(alpharight) &= left[frac{x^{alpha + 1}}{alpha + 1}cdotarcsin(x)right]_0^1 - int_0^1 frac{x^{alpha + 1}}{alpha + 1} cdot frac{1}{sqrt{1 - x^2}} :dx \
&= frac{pi}{2left(alpha + 1right)} - frac{1}{alpha + 1}int_0^1 x^{alpha + 1}left(1 - x^2right)^{-frac{1}{2}} :dx
end{align}



Here make the substitution $u = x^2$ to obtain



begin{align}
Fleft(alpharight) &= frac{pi}{2left(alpha + 1right)} - frac{1}{alpha + 1}int_0^1 left(sqrt{u}right)^{alpha + 1}left(1 - uright)^{-frac{1}{2}} frac{:du}{2sqrt{u}} \
&= frac{pi}{2left(alpha + 1right)} - frac{1}{2left(alpha + 1right)}int_0^1 u^{frac{alpha}{2}}left(1 - uright) ^{-frac{1}{2}} :du \
&= frac{1}{2left(alpha + 1right)} left[ pi - Bleft(frac{alpha + 2}{2}, frac{1}{2} right) right]
end{align}



begin{align}
Fleft(alpharight) &=frac{1}{2left(alpha + 1right)} left[ pi - frac{Gammaleft(frac{alpha + 2}{2}right)Gammaleft(frac{1}{2}right)}{Gammaleft(frac{alpha + 2}{2} + frac{1}{2}right)} right] \
&= frac{1}{2left(alpha + 1right)} left[ pi - frac{Gammaleft(frac{alpha + 2}{2}right)sqrt{pi}}{Gammaleft(frac{alpha + 3}{2}right) } right] \
&= frac{sqrt{pi}}{2left(alpha + 1right)} left[ sqrt{pi} - frac{Gammaleft(frac{alpha + 2}{2}right)}{Gammaleft(frac{alpha + 3}{2}right) } right]
end{align}



Edits:
Correction of original limit observation (now removed)
Correction of not stating region of convergence for $alpha$.
Correction of 1/sqrt to sqrt in final line.



Thanks to those commentators for pointing out.










share|cite|improve this question











$endgroup$








  • 1




    $begingroup$
    How would you expect to get $Gamma$'s out of Feynman Trick or whatever else? (And, BTW, $displaystylelim_{alphatoinfty}F(alpha)=0$).
    $endgroup$
    – metamorphy
    Dec 10 '18 at 1:03






  • 2




    $begingroup$
    There needs to be some restrictions on the value of $alpha$ otherwise the improper integral will not converge, namely $alpha > -2$. The case $alpha = -1$ also needs to be handled with care.
    $endgroup$
    – omegadot
    Dec 10 '18 at 2:12






  • 1




    $begingroup$
    Yes, sorry. I meant to say it only converges for $alpha > -2$ and diverges for $alpha leqslant -2$.
    $endgroup$
    – omegadot
    Dec 10 '18 at 3:02






  • 1




    $begingroup$
    I'm fairly certain that's the fastest way to do it
    $endgroup$
    – clathratus
    Dec 10 '18 at 3:29






  • 1




    $begingroup$
    The $frac{1}{sqrt{pi }}$ in the last line needs to be $sqrt{pi}$.
    $endgroup$
    – JimB
    Dec 10 '18 at 5:12














5












5








5


1



$begingroup$


I'm looking for different methods to solve the following integral.
$$ Fleft(alpharight) = int_{0}^{1} x^alpha arcsin(x):dx$$



For $alpha > 0$



Here the method I took was to employ integration by parts and then call to special functions, but can this equally be achieved with say a Feynman Trick? or another form integral transform?



My approach in detail:



Employ integration by parts:



begin{align}
v'(x) &= x^alpha & u(x) &= arcsin(x) \
v(x) &= frac{x^{alpha + 1}}{alpha + 1} & u'(x) &= frac{1}{sqrt{1 - x^2}}
end{align}



Thus,



begin{align}
Fleft(alpharight) &= left[frac{x^{alpha + 1}}{alpha + 1}cdotarcsin(x)right]_0^1 - int_0^1 frac{x^{alpha + 1}}{alpha + 1} cdot frac{1}{sqrt{1 - x^2}} :dx \
&= frac{pi}{2left(alpha + 1right)} - frac{1}{alpha + 1}int_0^1 x^{alpha + 1}left(1 - x^2right)^{-frac{1}{2}} :dx
end{align}



Here make the substitution $u = x^2$ to obtain



begin{align}
Fleft(alpharight) &= frac{pi}{2left(alpha + 1right)} - frac{1}{alpha + 1}int_0^1 left(sqrt{u}right)^{alpha + 1}left(1 - uright)^{-frac{1}{2}} frac{:du}{2sqrt{u}} \
&= frac{pi}{2left(alpha + 1right)} - frac{1}{2left(alpha + 1right)}int_0^1 u^{frac{alpha}{2}}left(1 - uright) ^{-frac{1}{2}} :du \
&= frac{1}{2left(alpha + 1right)} left[ pi - Bleft(frac{alpha + 2}{2}, frac{1}{2} right) right]
end{align}



begin{align}
Fleft(alpharight) &=frac{1}{2left(alpha + 1right)} left[ pi - frac{Gammaleft(frac{alpha + 2}{2}right)Gammaleft(frac{1}{2}right)}{Gammaleft(frac{alpha + 2}{2} + frac{1}{2}right)} right] \
&= frac{1}{2left(alpha + 1right)} left[ pi - frac{Gammaleft(frac{alpha + 2}{2}right)sqrt{pi}}{Gammaleft(frac{alpha + 3}{2}right) } right] \
&= frac{sqrt{pi}}{2left(alpha + 1right)} left[ sqrt{pi} - frac{Gammaleft(frac{alpha + 2}{2}right)}{Gammaleft(frac{alpha + 3}{2}right) } right]
end{align}



Edits:
Correction of original limit observation (now removed)
Correction of not stating region of convergence for $alpha$.
Correction of 1/sqrt to sqrt in final line.



Thanks to those commentators for pointing out.










share|cite|improve this question











$endgroup$




I'm looking for different methods to solve the following integral.
$$ Fleft(alpharight) = int_{0}^{1} x^alpha arcsin(x):dx$$



For $alpha > 0$



Here the method I took was to employ integration by parts and then call to special functions, but can this equally be achieved with say a Feynman Trick? or another form integral transform?



My approach in detail:



Employ integration by parts:



begin{align}
v'(x) &= x^alpha & u(x) &= arcsin(x) \
v(x) &= frac{x^{alpha + 1}}{alpha + 1} & u'(x) &= frac{1}{sqrt{1 - x^2}}
end{align}



Thus,



begin{align}
Fleft(alpharight) &= left[frac{x^{alpha + 1}}{alpha + 1}cdotarcsin(x)right]_0^1 - int_0^1 frac{x^{alpha + 1}}{alpha + 1} cdot frac{1}{sqrt{1 - x^2}} :dx \
&= frac{pi}{2left(alpha + 1right)} - frac{1}{alpha + 1}int_0^1 x^{alpha + 1}left(1 - x^2right)^{-frac{1}{2}} :dx
end{align}



Here make the substitution $u = x^2$ to obtain



begin{align}
Fleft(alpharight) &= frac{pi}{2left(alpha + 1right)} - frac{1}{alpha + 1}int_0^1 left(sqrt{u}right)^{alpha + 1}left(1 - uright)^{-frac{1}{2}} frac{:du}{2sqrt{u}} \
&= frac{pi}{2left(alpha + 1right)} - frac{1}{2left(alpha + 1right)}int_0^1 u^{frac{alpha}{2}}left(1 - uright) ^{-frac{1}{2}} :du \
&= frac{1}{2left(alpha + 1right)} left[ pi - Bleft(frac{alpha + 2}{2}, frac{1}{2} right) right]
end{align}



begin{align}
Fleft(alpharight) &=frac{1}{2left(alpha + 1right)} left[ pi - frac{Gammaleft(frac{alpha + 2}{2}right)Gammaleft(frac{1}{2}right)}{Gammaleft(frac{alpha + 2}{2} + frac{1}{2}right)} right] \
&= frac{1}{2left(alpha + 1right)} left[ pi - frac{Gammaleft(frac{alpha + 2}{2}right)sqrt{pi}}{Gammaleft(frac{alpha + 3}{2}right) } right] \
&= frac{sqrt{pi}}{2left(alpha + 1right)} left[ sqrt{pi} - frac{Gammaleft(frac{alpha + 2}{2}right)}{Gammaleft(frac{alpha + 3}{2}right) } right]
end{align}



Edits:
Correction of original limit observation (now removed)
Correction of not stating region of convergence for $alpha$.
Correction of 1/sqrt to sqrt in final line.



Thanks to those commentators for pointing out.







integration definite-integrals error-function beta-function gamma-distribution






share|cite|improve this question















share|cite|improve this question













share|cite|improve this question




share|cite|improve this question








edited Dec 10 '18 at 5:14







DavidG

















asked Dec 10 '18 at 0:37









DavidGDavidG

1




1








  • 1




    $begingroup$
    How would you expect to get $Gamma$'s out of Feynman Trick or whatever else? (And, BTW, $displaystylelim_{alphatoinfty}F(alpha)=0$).
    $endgroup$
    – metamorphy
    Dec 10 '18 at 1:03






  • 2




    $begingroup$
    There needs to be some restrictions on the value of $alpha$ otherwise the improper integral will not converge, namely $alpha > -2$. The case $alpha = -1$ also needs to be handled with care.
    $endgroup$
    – omegadot
    Dec 10 '18 at 2:12






  • 1




    $begingroup$
    Yes, sorry. I meant to say it only converges for $alpha > -2$ and diverges for $alpha leqslant -2$.
    $endgroup$
    – omegadot
    Dec 10 '18 at 3:02






  • 1




    $begingroup$
    I'm fairly certain that's the fastest way to do it
    $endgroup$
    – clathratus
    Dec 10 '18 at 3:29






  • 1




    $begingroup$
    The $frac{1}{sqrt{pi }}$ in the last line needs to be $sqrt{pi}$.
    $endgroup$
    – JimB
    Dec 10 '18 at 5:12














  • 1




    $begingroup$
    How would you expect to get $Gamma$'s out of Feynman Trick or whatever else? (And, BTW, $displaystylelim_{alphatoinfty}F(alpha)=0$).
    $endgroup$
    – metamorphy
    Dec 10 '18 at 1:03






  • 2




    $begingroup$
    There needs to be some restrictions on the value of $alpha$ otherwise the improper integral will not converge, namely $alpha > -2$. The case $alpha = -1$ also needs to be handled with care.
    $endgroup$
    – omegadot
    Dec 10 '18 at 2:12






  • 1




    $begingroup$
    Yes, sorry. I meant to say it only converges for $alpha > -2$ and diverges for $alpha leqslant -2$.
    $endgroup$
    – omegadot
    Dec 10 '18 at 3:02






  • 1




    $begingroup$
    I'm fairly certain that's the fastest way to do it
    $endgroup$
    – clathratus
    Dec 10 '18 at 3:29






  • 1




    $begingroup$
    The $frac{1}{sqrt{pi }}$ in the last line needs to be $sqrt{pi}$.
    $endgroup$
    – JimB
    Dec 10 '18 at 5:12








1




1




$begingroup$
How would you expect to get $Gamma$'s out of Feynman Trick or whatever else? (And, BTW, $displaystylelim_{alphatoinfty}F(alpha)=0$).
$endgroup$
– metamorphy
Dec 10 '18 at 1:03




$begingroup$
How would you expect to get $Gamma$'s out of Feynman Trick or whatever else? (And, BTW, $displaystylelim_{alphatoinfty}F(alpha)=0$).
$endgroup$
– metamorphy
Dec 10 '18 at 1:03




2




2




$begingroup$
There needs to be some restrictions on the value of $alpha$ otherwise the improper integral will not converge, namely $alpha > -2$. The case $alpha = -1$ also needs to be handled with care.
$endgroup$
– omegadot
Dec 10 '18 at 2:12




$begingroup$
There needs to be some restrictions on the value of $alpha$ otherwise the improper integral will not converge, namely $alpha > -2$. The case $alpha = -1$ also needs to be handled with care.
$endgroup$
– omegadot
Dec 10 '18 at 2:12




1




1




$begingroup$
Yes, sorry. I meant to say it only converges for $alpha > -2$ and diverges for $alpha leqslant -2$.
$endgroup$
– omegadot
Dec 10 '18 at 3:02




$begingroup$
Yes, sorry. I meant to say it only converges for $alpha > -2$ and diverges for $alpha leqslant -2$.
$endgroup$
– omegadot
Dec 10 '18 at 3:02




1




1




$begingroup$
I'm fairly certain that's the fastest way to do it
$endgroup$
– clathratus
Dec 10 '18 at 3:29




$begingroup$
I'm fairly certain that's the fastest way to do it
$endgroup$
– clathratus
Dec 10 '18 at 3:29




1




1




$begingroup$
The $frac{1}{sqrt{pi }}$ in the last line needs to be $sqrt{pi}$.
$endgroup$
– JimB
Dec 10 '18 at 5:12




$begingroup$
The $frac{1}{sqrt{pi }}$ in the last line needs to be $sqrt{pi}$.
$endgroup$
– JimB
Dec 10 '18 at 5:12










2 Answers
2






active

oldest

votes


















2












$begingroup$

Here is a method that relies on using a double integral.



Noting the integral converges for $alpha > -2$, recognising
$$arcsin x = int_0^x frac{du}{sqrt{1 - u^2}},$$
the integral can be rewritten as
$$I = int_0^1 int_0^x frac{x^alpha}{sqrt{1 - u^2}} , du dx.$$
On changing the order of integration, one has
begin{equation}
int_0^1 int_u^1 frac{x^alpha}{sqrt{1 - u^2}} , dx du. qquad (*)
end{equation}

After performing the $x$-integral we are left with
$$I = frac{1}{alpha + 1} int_0^1 frac{1 - u^{alpha + 1}}{sqrt{1 - u^2}} , du, quad alpha neq -1.$$
Enforcing a substitution of $u mapsto sqrt{u}$ results in
$$I = frac{1}{2(alpha + 1)} int_0^1 left (frac{1}{sqrt{u(1 - u)}} - frac{u^{alpha/2}}{sqrt{ 1 - u}} right ) , du = I_1 - I_2.$$



The first of the integrals is trivial
$$I_1 = frac{1}{2(alpha + 1)} int_0^1 frac{du}{sqrt{frac{1}{4} - (u - frac{1}{2})^2}} = frac{1}{2(alpha + 1)} arcsin (2u - 1) Big{|}^1_0 = frac{pi}{2(alpha + 1)}.$$



For the second of the integrals, it can be evaluated by writing it in terms of the beta function. Here
begin{align}
I_2 &= int_0^1 frac{u^{alpha/2}}{sqrt{1 - u}} , du\
&= int_0^1 u^{(alpha/2 + 1) - 1} (1 - u)^{1/2 - 1} , du\
&= text{B} left (frac{alpha}{2} + 1, frac{1}{2} right )\
&= sqrt{pi} , frac{Gamma left (frac{alpha + 2}{2} right )}{Gamma left (frac{alpha + 3}{2} right )}.\
end{align}

Thus
$$I = frac{1}{2(alpha + 1)} left [pi - sqrt{pi} , frac{Gamma left (frac{alpha + 2}{2} right )}{Gamma left (frac{alpha + 3}{2} right )} right ], qquad alpha neq -1.$$



For the case when $alpha = -1$, the double integral at ($*$) becomes
$$I = int_0^1 int_u^1 frac{1}{xsqrt{1 - u^2}} , dx du.$$
After performing the $x$-integral which yields a natural logarithm, one has
$$I = -int_0^1 frac{ln u}{sqrt{1 - u^2}} , du.$$
Enforcing a substitution of $u mapsto sin u$ leads to
$$I = -int_0^{pi/2} ln (sin u) , du. qquad (**)$$
The integral appearing in ($**$) is quite famous and reasonably (?) well known. Its evaluation can be found either here or here. Thus
$$I = frac{pi}{2} ln 2, qquad alpha = -1.$$






share|cite|improve this answer









$endgroup$













  • $begingroup$
    A fantastic solution. Thank you very much very. I'm been working on developing my general nasty integral skills and this mode of converting a single to double integral is great. Is this a form of Feynman's Trick? Regardless, again, fantastic solution.
    $endgroup$
    – DavidG
    Dec 10 '18 at 12:10



















1












$begingroup$

Answer 2.0:



We know that for $|x|<1$,
$$arcsin x=sum_{kgeq0}frac{(1/2)_k}{k!(2k+1)}x^{2k+1}$$
Where $$(1/2)_k=frac{Gamma(1/2+k)}{Gamma(1/2)}$$
Hence we may begin with
$$F(a)=sum_{kgeq0}frac{(1/2)_k}{k!(2k+1)}int_0^1x^{2k+a+1}mathrm dx=sum_{kgeq0}frac{(1/2)_k}{k!(2k+1)(2k+2+a)}$$
Then we play with the fractions a little to get
$$F(a)=frac1{a+1}sum_{kgeq0}frac{(1/2)_k}{k!}bigg[frac1{2k+1}-frac1{2k+2+a}bigg]$$
$$F(a)=frac1{a+1}sum_{ngeq0}frac{(1/2)_n}{n!}frac1{2n+1}-frac1{a+1}sum_{kgeq0}frac{(1/2)_k}{k!}frac1{2k+2+a}$$
$$F(a)=fracpi{2(a+1)}-frac1{a+1}sum_{kgeq0}frac{(1/2)_k}{k!(2k+2+a)}$$
$$F(a)=fracpi{2(a+1)}-frac1{a+1}sum_{kgeq0}frac{(1/2)_k}{k!}int_0^1x^{2k+1+a}mathrm dx$$
$$F(a)=fracpi{2(a+1)}-frac1{a+1}int_0^1sum_{kgeq0}frac{(1/2)_k}{k!}x^{2k+1+a}mathrm dx$$
$$F(a)=fracpi{2(a+1)}-frac1{a+1}int_0^1frac{x^{a+1}}{Gamma(1/2)}sum_{kgeq0}frac{Gamma(1/2+k)}{k!}x^{2k}mathrm dx$$
$$F(a)=fracpi{2(a+1)}-frac1{a+1}int_0^1frac{x^{a+1}}{sqrt{1-x^2}}mathrm dx$$
$u=x^2$:
$$F(a)=fracpi{2(a+1)}-frac1{2(a+1)}int_0^1u^{a/2}(1-u)^{-1/2}mathrm du$$
$$F(a)=fracpi{2(a+1)}-frac{Gamma(frac{a+2}2)Gamma(frac12)}{2(a+1)Gamma(frac{a+3}2)}$$
$$F(a)=frac{sqrt{pi}}{2(a+1)}bigg[sqrt{pi}-frac{Gamma(frac{a+2}2)}{Gamma(frac{a+3}2)}bigg]$$






share|cite|improve this answer









$endgroup$













    Your Answer





    StackExchange.ifUsing("editor", function () {
    return StackExchange.using("mathjaxEditing", function () {
    StackExchange.MarkdownEditor.creationCallbacks.add(function (editor, postfix) {
    StackExchange.mathjaxEditing.prepareWmdForMathJax(editor, postfix, [["$", "$"], ["\\(","\\)"]]);
    });
    });
    }, "mathjax-editing");

    StackExchange.ready(function() {
    var channelOptions = {
    tags: "".split(" "),
    id: "69"
    };
    initTagRenderer("".split(" "), "".split(" "), channelOptions);

    StackExchange.using("externalEditor", function() {
    // Have to fire editor after snippets, if snippets enabled
    if (StackExchange.settings.snippets.snippetsEnabled) {
    StackExchange.using("snippets", function() {
    createEditor();
    });
    }
    else {
    createEditor();
    }
    });

    function createEditor() {
    StackExchange.prepareEditor({
    heartbeatType: 'answer',
    autoActivateHeartbeat: false,
    convertImagesToLinks: true,
    noModals: true,
    showLowRepImageUploadWarning: true,
    reputationToPostImages: 10,
    bindNavPrevention: true,
    postfix: "",
    imageUploader: {
    brandingHtml: "Powered by u003ca class="icon-imgur-white" href="https://imgur.com/"u003eu003c/au003e",
    contentPolicyHtml: "User contributions licensed under u003ca href="https://creativecommons.org/licenses/by-sa/3.0/"u003ecc by-sa 3.0 with attribution requiredu003c/au003e u003ca href="https://stackoverflow.com/legal/content-policy"u003e(content policy)u003c/au003e",
    allowUrls: true
    },
    noCode: true, onDemand: true,
    discardSelector: ".discard-answer"
    ,immediatelyShowMarkdownHelp:true
    });


    }
    });














    draft saved

    draft discarded


















    StackExchange.ready(
    function () {
    StackExchange.openid.initPostLogin('.new-post-login', 'https%3a%2f%2fmath.stackexchange.com%2fquestions%2f3033251%2fseeking-methods-to-solve-f-left-alpha-right-int-01-x-alpha-arcsinx%23new-answer', 'question_page');
    }
    );

    Post as a guest















    Required, but never shown

























    2 Answers
    2






    active

    oldest

    votes








    2 Answers
    2






    active

    oldest

    votes









    active

    oldest

    votes






    active

    oldest

    votes









    2












    $begingroup$

    Here is a method that relies on using a double integral.



    Noting the integral converges for $alpha > -2$, recognising
    $$arcsin x = int_0^x frac{du}{sqrt{1 - u^2}},$$
    the integral can be rewritten as
    $$I = int_0^1 int_0^x frac{x^alpha}{sqrt{1 - u^2}} , du dx.$$
    On changing the order of integration, one has
    begin{equation}
    int_0^1 int_u^1 frac{x^alpha}{sqrt{1 - u^2}} , dx du. qquad (*)
    end{equation}

    After performing the $x$-integral we are left with
    $$I = frac{1}{alpha + 1} int_0^1 frac{1 - u^{alpha + 1}}{sqrt{1 - u^2}} , du, quad alpha neq -1.$$
    Enforcing a substitution of $u mapsto sqrt{u}$ results in
    $$I = frac{1}{2(alpha + 1)} int_0^1 left (frac{1}{sqrt{u(1 - u)}} - frac{u^{alpha/2}}{sqrt{ 1 - u}} right ) , du = I_1 - I_2.$$



    The first of the integrals is trivial
    $$I_1 = frac{1}{2(alpha + 1)} int_0^1 frac{du}{sqrt{frac{1}{4} - (u - frac{1}{2})^2}} = frac{1}{2(alpha + 1)} arcsin (2u - 1) Big{|}^1_0 = frac{pi}{2(alpha + 1)}.$$



    For the second of the integrals, it can be evaluated by writing it in terms of the beta function. Here
    begin{align}
    I_2 &= int_0^1 frac{u^{alpha/2}}{sqrt{1 - u}} , du\
    &= int_0^1 u^{(alpha/2 + 1) - 1} (1 - u)^{1/2 - 1} , du\
    &= text{B} left (frac{alpha}{2} + 1, frac{1}{2} right )\
    &= sqrt{pi} , frac{Gamma left (frac{alpha + 2}{2} right )}{Gamma left (frac{alpha + 3}{2} right )}.\
    end{align}

    Thus
    $$I = frac{1}{2(alpha + 1)} left [pi - sqrt{pi} , frac{Gamma left (frac{alpha + 2}{2} right )}{Gamma left (frac{alpha + 3}{2} right )} right ], qquad alpha neq -1.$$



    For the case when $alpha = -1$, the double integral at ($*$) becomes
    $$I = int_0^1 int_u^1 frac{1}{xsqrt{1 - u^2}} , dx du.$$
    After performing the $x$-integral which yields a natural logarithm, one has
    $$I = -int_0^1 frac{ln u}{sqrt{1 - u^2}} , du.$$
    Enforcing a substitution of $u mapsto sin u$ leads to
    $$I = -int_0^{pi/2} ln (sin u) , du. qquad (**)$$
    The integral appearing in ($**$) is quite famous and reasonably (?) well known. Its evaluation can be found either here or here. Thus
    $$I = frac{pi}{2} ln 2, qquad alpha = -1.$$






    share|cite|improve this answer









    $endgroup$













    • $begingroup$
      A fantastic solution. Thank you very much very. I'm been working on developing my general nasty integral skills and this mode of converting a single to double integral is great. Is this a form of Feynman's Trick? Regardless, again, fantastic solution.
      $endgroup$
      – DavidG
      Dec 10 '18 at 12:10
















    2












    $begingroup$

    Here is a method that relies on using a double integral.



    Noting the integral converges for $alpha > -2$, recognising
    $$arcsin x = int_0^x frac{du}{sqrt{1 - u^2}},$$
    the integral can be rewritten as
    $$I = int_0^1 int_0^x frac{x^alpha}{sqrt{1 - u^2}} , du dx.$$
    On changing the order of integration, one has
    begin{equation}
    int_0^1 int_u^1 frac{x^alpha}{sqrt{1 - u^2}} , dx du. qquad (*)
    end{equation}

    After performing the $x$-integral we are left with
    $$I = frac{1}{alpha + 1} int_0^1 frac{1 - u^{alpha + 1}}{sqrt{1 - u^2}} , du, quad alpha neq -1.$$
    Enforcing a substitution of $u mapsto sqrt{u}$ results in
    $$I = frac{1}{2(alpha + 1)} int_0^1 left (frac{1}{sqrt{u(1 - u)}} - frac{u^{alpha/2}}{sqrt{ 1 - u}} right ) , du = I_1 - I_2.$$



    The first of the integrals is trivial
    $$I_1 = frac{1}{2(alpha + 1)} int_0^1 frac{du}{sqrt{frac{1}{4} - (u - frac{1}{2})^2}} = frac{1}{2(alpha + 1)} arcsin (2u - 1) Big{|}^1_0 = frac{pi}{2(alpha + 1)}.$$



    For the second of the integrals, it can be evaluated by writing it in terms of the beta function. Here
    begin{align}
    I_2 &= int_0^1 frac{u^{alpha/2}}{sqrt{1 - u}} , du\
    &= int_0^1 u^{(alpha/2 + 1) - 1} (1 - u)^{1/2 - 1} , du\
    &= text{B} left (frac{alpha}{2} + 1, frac{1}{2} right )\
    &= sqrt{pi} , frac{Gamma left (frac{alpha + 2}{2} right )}{Gamma left (frac{alpha + 3}{2} right )}.\
    end{align}

    Thus
    $$I = frac{1}{2(alpha + 1)} left [pi - sqrt{pi} , frac{Gamma left (frac{alpha + 2}{2} right )}{Gamma left (frac{alpha + 3}{2} right )} right ], qquad alpha neq -1.$$



    For the case when $alpha = -1$, the double integral at ($*$) becomes
    $$I = int_0^1 int_u^1 frac{1}{xsqrt{1 - u^2}} , dx du.$$
    After performing the $x$-integral which yields a natural logarithm, one has
    $$I = -int_0^1 frac{ln u}{sqrt{1 - u^2}} , du.$$
    Enforcing a substitution of $u mapsto sin u$ leads to
    $$I = -int_0^{pi/2} ln (sin u) , du. qquad (**)$$
    The integral appearing in ($**$) is quite famous and reasonably (?) well known. Its evaluation can be found either here or here. Thus
    $$I = frac{pi}{2} ln 2, qquad alpha = -1.$$






    share|cite|improve this answer









    $endgroup$













    • $begingroup$
      A fantastic solution. Thank you very much very. I'm been working on developing my general nasty integral skills and this mode of converting a single to double integral is great. Is this a form of Feynman's Trick? Regardless, again, fantastic solution.
      $endgroup$
      – DavidG
      Dec 10 '18 at 12:10














    2












    2








    2





    $begingroup$

    Here is a method that relies on using a double integral.



    Noting the integral converges for $alpha > -2$, recognising
    $$arcsin x = int_0^x frac{du}{sqrt{1 - u^2}},$$
    the integral can be rewritten as
    $$I = int_0^1 int_0^x frac{x^alpha}{sqrt{1 - u^2}} , du dx.$$
    On changing the order of integration, one has
    begin{equation}
    int_0^1 int_u^1 frac{x^alpha}{sqrt{1 - u^2}} , dx du. qquad (*)
    end{equation}

    After performing the $x$-integral we are left with
    $$I = frac{1}{alpha + 1} int_0^1 frac{1 - u^{alpha + 1}}{sqrt{1 - u^2}} , du, quad alpha neq -1.$$
    Enforcing a substitution of $u mapsto sqrt{u}$ results in
    $$I = frac{1}{2(alpha + 1)} int_0^1 left (frac{1}{sqrt{u(1 - u)}} - frac{u^{alpha/2}}{sqrt{ 1 - u}} right ) , du = I_1 - I_2.$$



    The first of the integrals is trivial
    $$I_1 = frac{1}{2(alpha + 1)} int_0^1 frac{du}{sqrt{frac{1}{4} - (u - frac{1}{2})^2}} = frac{1}{2(alpha + 1)} arcsin (2u - 1) Big{|}^1_0 = frac{pi}{2(alpha + 1)}.$$



    For the second of the integrals, it can be evaluated by writing it in terms of the beta function. Here
    begin{align}
    I_2 &= int_0^1 frac{u^{alpha/2}}{sqrt{1 - u}} , du\
    &= int_0^1 u^{(alpha/2 + 1) - 1} (1 - u)^{1/2 - 1} , du\
    &= text{B} left (frac{alpha}{2} + 1, frac{1}{2} right )\
    &= sqrt{pi} , frac{Gamma left (frac{alpha + 2}{2} right )}{Gamma left (frac{alpha + 3}{2} right )}.\
    end{align}

    Thus
    $$I = frac{1}{2(alpha + 1)} left [pi - sqrt{pi} , frac{Gamma left (frac{alpha + 2}{2} right )}{Gamma left (frac{alpha + 3}{2} right )} right ], qquad alpha neq -1.$$



    For the case when $alpha = -1$, the double integral at ($*$) becomes
    $$I = int_0^1 int_u^1 frac{1}{xsqrt{1 - u^2}} , dx du.$$
    After performing the $x$-integral which yields a natural logarithm, one has
    $$I = -int_0^1 frac{ln u}{sqrt{1 - u^2}} , du.$$
    Enforcing a substitution of $u mapsto sin u$ leads to
    $$I = -int_0^{pi/2} ln (sin u) , du. qquad (**)$$
    The integral appearing in ($**$) is quite famous and reasonably (?) well known. Its evaluation can be found either here or here. Thus
    $$I = frac{pi}{2} ln 2, qquad alpha = -1.$$






    share|cite|improve this answer









    $endgroup$



    Here is a method that relies on using a double integral.



    Noting the integral converges for $alpha > -2$, recognising
    $$arcsin x = int_0^x frac{du}{sqrt{1 - u^2}},$$
    the integral can be rewritten as
    $$I = int_0^1 int_0^x frac{x^alpha}{sqrt{1 - u^2}} , du dx.$$
    On changing the order of integration, one has
    begin{equation}
    int_0^1 int_u^1 frac{x^alpha}{sqrt{1 - u^2}} , dx du. qquad (*)
    end{equation}

    After performing the $x$-integral we are left with
    $$I = frac{1}{alpha + 1} int_0^1 frac{1 - u^{alpha + 1}}{sqrt{1 - u^2}} , du, quad alpha neq -1.$$
    Enforcing a substitution of $u mapsto sqrt{u}$ results in
    $$I = frac{1}{2(alpha + 1)} int_0^1 left (frac{1}{sqrt{u(1 - u)}} - frac{u^{alpha/2}}{sqrt{ 1 - u}} right ) , du = I_1 - I_2.$$



    The first of the integrals is trivial
    $$I_1 = frac{1}{2(alpha + 1)} int_0^1 frac{du}{sqrt{frac{1}{4} - (u - frac{1}{2})^2}} = frac{1}{2(alpha + 1)} arcsin (2u - 1) Big{|}^1_0 = frac{pi}{2(alpha + 1)}.$$



    For the second of the integrals, it can be evaluated by writing it in terms of the beta function. Here
    begin{align}
    I_2 &= int_0^1 frac{u^{alpha/2}}{sqrt{1 - u}} , du\
    &= int_0^1 u^{(alpha/2 + 1) - 1} (1 - u)^{1/2 - 1} , du\
    &= text{B} left (frac{alpha}{2} + 1, frac{1}{2} right )\
    &= sqrt{pi} , frac{Gamma left (frac{alpha + 2}{2} right )}{Gamma left (frac{alpha + 3}{2} right )}.\
    end{align}

    Thus
    $$I = frac{1}{2(alpha + 1)} left [pi - sqrt{pi} , frac{Gamma left (frac{alpha + 2}{2} right )}{Gamma left (frac{alpha + 3}{2} right )} right ], qquad alpha neq -1.$$



    For the case when $alpha = -1$, the double integral at ($*$) becomes
    $$I = int_0^1 int_u^1 frac{1}{xsqrt{1 - u^2}} , dx du.$$
    After performing the $x$-integral which yields a natural logarithm, one has
    $$I = -int_0^1 frac{ln u}{sqrt{1 - u^2}} , du.$$
    Enforcing a substitution of $u mapsto sin u$ leads to
    $$I = -int_0^{pi/2} ln (sin u) , du. qquad (**)$$
    The integral appearing in ($**$) is quite famous and reasonably (?) well known. Its evaluation can be found either here or here. Thus
    $$I = frac{pi}{2} ln 2, qquad alpha = -1.$$







    share|cite|improve this answer












    share|cite|improve this answer



    share|cite|improve this answer










    answered Dec 10 '18 at 9:01









    omegadotomegadot

    6,4342829




    6,4342829












    • $begingroup$
      A fantastic solution. Thank you very much very. I'm been working on developing my general nasty integral skills and this mode of converting a single to double integral is great. Is this a form of Feynman's Trick? Regardless, again, fantastic solution.
      $endgroup$
      – DavidG
      Dec 10 '18 at 12:10


















    • $begingroup$
      A fantastic solution. Thank you very much very. I'm been working on developing my general nasty integral skills and this mode of converting a single to double integral is great. Is this a form of Feynman's Trick? Regardless, again, fantastic solution.
      $endgroup$
      – DavidG
      Dec 10 '18 at 12:10
















    $begingroup$
    A fantastic solution. Thank you very much very. I'm been working on developing my general nasty integral skills and this mode of converting a single to double integral is great. Is this a form of Feynman's Trick? Regardless, again, fantastic solution.
    $endgroup$
    – DavidG
    Dec 10 '18 at 12:10




    $begingroup$
    A fantastic solution. Thank you very much very. I'm been working on developing my general nasty integral skills and this mode of converting a single to double integral is great. Is this a form of Feynman's Trick? Regardless, again, fantastic solution.
    $endgroup$
    – DavidG
    Dec 10 '18 at 12:10











    1












    $begingroup$

    Answer 2.0:



    We know that for $|x|<1$,
    $$arcsin x=sum_{kgeq0}frac{(1/2)_k}{k!(2k+1)}x^{2k+1}$$
    Where $$(1/2)_k=frac{Gamma(1/2+k)}{Gamma(1/2)}$$
    Hence we may begin with
    $$F(a)=sum_{kgeq0}frac{(1/2)_k}{k!(2k+1)}int_0^1x^{2k+a+1}mathrm dx=sum_{kgeq0}frac{(1/2)_k}{k!(2k+1)(2k+2+a)}$$
    Then we play with the fractions a little to get
    $$F(a)=frac1{a+1}sum_{kgeq0}frac{(1/2)_k}{k!}bigg[frac1{2k+1}-frac1{2k+2+a}bigg]$$
    $$F(a)=frac1{a+1}sum_{ngeq0}frac{(1/2)_n}{n!}frac1{2n+1}-frac1{a+1}sum_{kgeq0}frac{(1/2)_k}{k!}frac1{2k+2+a}$$
    $$F(a)=fracpi{2(a+1)}-frac1{a+1}sum_{kgeq0}frac{(1/2)_k}{k!(2k+2+a)}$$
    $$F(a)=fracpi{2(a+1)}-frac1{a+1}sum_{kgeq0}frac{(1/2)_k}{k!}int_0^1x^{2k+1+a}mathrm dx$$
    $$F(a)=fracpi{2(a+1)}-frac1{a+1}int_0^1sum_{kgeq0}frac{(1/2)_k}{k!}x^{2k+1+a}mathrm dx$$
    $$F(a)=fracpi{2(a+1)}-frac1{a+1}int_0^1frac{x^{a+1}}{Gamma(1/2)}sum_{kgeq0}frac{Gamma(1/2+k)}{k!}x^{2k}mathrm dx$$
    $$F(a)=fracpi{2(a+1)}-frac1{a+1}int_0^1frac{x^{a+1}}{sqrt{1-x^2}}mathrm dx$$
    $u=x^2$:
    $$F(a)=fracpi{2(a+1)}-frac1{2(a+1)}int_0^1u^{a/2}(1-u)^{-1/2}mathrm du$$
    $$F(a)=fracpi{2(a+1)}-frac{Gamma(frac{a+2}2)Gamma(frac12)}{2(a+1)Gamma(frac{a+3}2)}$$
    $$F(a)=frac{sqrt{pi}}{2(a+1)}bigg[sqrt{pi}-frac{Gamma(frac{a+2}2)}{Gamma(frac{a+3}2)}bigg]$$






    share|cite|improve this answer









    $endgroup$


















      1












      $begingroup$

      Answer 2.0:



      We know that for $|x|<1$,
      $$arcsin x=sum_{kgeq0}frac{(1/2)_k}{k!(2k+1)}x^{2k+1}$$
      Where $$(1/2)_k=frac{Gamma(1/2+k)}{Gamma(1/2)}$$
      Hence we may begin with
      $$F(a)=sum_{kgeq0}frac{(1/2)_k}{k!(2k+1)}int_0^1x^{2k+a+1}mathrm dx=sum_{kgeq0}frac{(1/2)_k}{k!(2k+1)(2k+2+a)}$$
      Then we play with the fractions a little to get
      $$F(a)=frac1{a+1}sum_{kgeq0}frac{(1/2)_k}{k!}bigg[frac1{2k+1}-frac1{2k+2+a}bigg]$$
      $$F(a)=frac1{a+1}sum_{ngeq0}frac{(1/2)_n}{n!}frac1{2n+1}-frac1{a+1}sum_{kgeq0}frac{(1/2)_k}{k!}frac1{2k+2+a}$$
      $$F(a)=fracpi{2(a+1)}-frac1{a+1}sum_{kgeq0}frac{(1/2)_k}{k!(2k+2+a)}$$
      $$F(a)=fracpi{2(a+1)}-frac1{a+1}sum_{kgeq0}frac{(1/2)_k}{k!}int_0^1x^{2k+1+a}mathrm dx$$
      $$F(a)=fracpi{2(a+1)}-frac1{a+1}int_0^1sum_{kgeq0}frac{(1/2)_k}{k!}x^{2k+1+a}mathrm dx$$
      $$F(a)=fracpi{2(a+1)}-frac1{a+1}int_0^1frac{x^{a+1}}{Gamma(1/2)}sum_{kgeq0}frac{Gamma(1/2+k)}{k!}x^{2k}mathrm dx$$
      $$F(a)=fracpi{2(a+1)}-frac1{a+1}int_0^1frac{x^{a+1}}{sqrt{1-x^2}}mathrm dx$$
      $u=x^2$:
      $$F(a)=fracpi{2(a+1)}-frac1{2(a+1)}int_0^1u^{a/2}(1-u)^{-1/2}mathrm du$$
      $$F(a)=fracpi{2(a+1)}-frac{Gamma(frac{a+2}2)Gamma(frac12)}{2(a+1)Gamma(frac{a+3}2)}$$
      $$F(a)=frac{sqrt{pi}}{2(a+1)}bigg[sqrt{pi}-frac{Gamma(frac{a+2}2)}{Gamma(frac{a+3}2)}bigg]$$






      share|cite|improve this answer









      $endgroup$
















        1












        1








        1





        $begingroup$

        Answer 2.0:



        We know that for $|x|<1$,
        $$arcsin x=sum_{kgeq0}frac{(1/2)_k}{k!(2k+1)}x^{2k+1}$$
        Where $$(1/2)_k=frac{Gamma(1/2+k)}{Gamma(1/2)}$$
        Hence we may begin with
        $$F(a)=sum_{kgeq0}frac{(1/2)_k}{k!(2k+1)}int_0^1x^{2k+a+1}mathrm dx=sum_{kgeq0}frac{(1/2)_k}{k!(2k+1)(2k+2+a)}$$
        Then we play with the fractions a little to get
        $$F(a)=frac1{a+1}sum_{kgeq0}frac{(1/2)_k}{k!}bigg[frac1{2k+1}-frac1{2k+2+a}bigg]$$
        $$F(a)=frac1{a+1}sum_{ngeq0}frac{(1/2)_n}{n!}frac1{2n+1}-frac1{a+1}sum_{kgeq0}frac{(1/2)_k}{k!}frac1{2k+2+a}$$
        $$F(a)=fracpi{2(a+1)}-frac1{a+1}sum_{kgeq0}frac{(1/2)_k}{k!(2k+2+a)}$$
        $$F(a)=fracpi{2(a+1)}-frac1{a+1}sum_{kgeq0}frac{(1/2)_k}{k!}int_0^1x^{2k+1+a}mathrm dx$$
        $$F(a)=fracpi{2(a+1)}-frac1{a+1}int_0^1sum_{kgeq0}frac{(1/2)_k}{k!}x^{2k+1+a}mathrm dx$$
        $$F(a)=fracpi{2(a+1)}-frac1{a+1}int_0^1frac{x^{a+1}}{Gamma(1/2)}sum_{kgeq0}frac{Gamma(1/2+k)}{k!}x^{2k}mathrm dx$$
        $$F(a)=fracpi{2(a+1)}-frac1{a+1}int_0^1frac{x^{a+1}}{sqrt{1-x^2}}mathrm dx$$
        $u=x^2$:
        $$F(a)=fracpi{2(a+1)}-frac1{2(a+1)}int_0^1u^{a/2}(1-u)^{-1/2}mathrm du$$
        $$F(a)=fracpi{2(a+1)}-frac{Gamma(frac{a+2}2)Gamma(frac12)}{2(a+1)Gamma(frac{a+3}2)}$$
        $$F(a)=frac{sqrt{pi}}{2(a+1)}bigg[sqrt{pi}-frac{Gamma(frac{a+2}2)}{Gamma(frac{a+3}2)}bigg]$$






        share|cite|improve this answer









        $endgroup$



        Answer 2.0:



        We know that for $|x|<1$,
        $$arcsin x=sum_{kgeq0}frac{(1/2)_k}{k!(2k+1)}x^{2k+1}$$
        Where $$(1/2)_k=frac{Gamma(1/2+k)}{Gamma(1/2)}$$
        Hence we may begin with
        $$F(a)=sum_{kgeq0}frac{(1/2)_k}{k!(2k+1)}int_0^1x^{2k+a+1}mathrm dx=sum_{kgeq0}frac{(1/2)_k}{k!(2k+1)(2k+2+a)}$$
        Then we play with the fractions a little to get
        $$F(a)=frac1{a+1}sum_{kgeq0}frac{(1/2)_k}{k!}bigg[frac1{2k+1}-frac1{2k+2+a}bigg]$$
        $$F(a)=frac1{a+1}sum_{ngeq0}frac{(1/2)_n}{n!}frac1{2n+1}-frac1{a+1}sum_{kgeq0}frac{(1/2)_k}{k!}frac1{2k+2+a}$$
        $$F(a)=fracpi{2(a+1)}-frac1{a+1}sum_{kgeq0}frac{(1/2)_k}{k!(2k+2+a)}$$
        $$F(a)=fracpi{2(a+1)}-frac1{a+1}sum_{kgeq0}frac{(1/2)_k}{k!}int_0^1x^{2k+1+a}mathrm dx$$
        $$F(a)=fracpi{2(a+1)}-frac1{a+1}int_0^1sum_{kgeq0}frac{(1/2)_k}{k!}x^{2k+1+a}mathrm dx$$
        $$F(a)=fracpi{2(a+1)}-frac1{a+1}int_0^1frac{x^{a+1}}{Gamma(1/2)}sum_{kgeq0}frac{Gamma(1/2+k)}{k!}x^{2k}mathrm dx$$
        $$F(a)=fracpi{2(a+1)}-frac1{a+1}int_0^1frac{x^{a+1}}{sqrt{1-x^2}}mathrm dx$$
        $u=x^2$:
        $$F(a)=fracpi{2(a+1)}-frac1{2(a+1)}int_0^1u^{a/2}(1-u)^{-1/2}mathrm du$$
        $$F(a)=fracpi{2(a+1)}-frac{Gamma(frac{a+2}2)Gamma(frac12)}{2(a+1)Gamma(frac{a+3}2)}$$
        $$F(a)=frac{sqrt{pi}}{2(a+1)}bigg[sqrt{pi}-frac{Gamma(frac{a+2}2)}{Gamma(frac{a+3}2)}bigg]$$







        share|cite|improve this answer












        share|cite|improve this answer



        share|cite|improve this answer










        answered Dec 10 '18 at 17:28









        clathratusclathratus

        5,1701338




        5,1701338






























            draft saved

            draft discarded




















































            Thanks for contributing an answer to Mathematics Stack Exchange!


            • Please be sure to answer the question. Provide details and share your research!

            But avoid



            • Asking for help, clarification, or responding to other answers.

            • Making statements based on opinion; back them up with references or personal experience.


            Use MathJax to format equations. MathJax reference.


            To learn more, see our tips on writing great answers.




            draft saved


            draft discarded














            StackExchange.ready(
            function () {
            StackExchange.openid.initPostLogin('.new-post-login', 'https%3a%2f%2fmath.stackexchange.com%2fquestions%2f3033251%2fseeking-methods-to-solve-f-left-alpha-right-int-01-x-alpha-arcsinx%23new-answer', 'question_page');
            }
            );

            Post as a guest















            Required, but never shown





















































            Required, but never shown














            Required, but never shown












            Required, but never shown







            Required, but never shown

































            Required, but never shown














            Required, but never shown












            Required, but never shown







            Required, but never shown







            Popular posts from this blog

            Biblatex bibliography style without URLs when DOI exists (in Overleaf with Zotero bibliography)

            ComboBox Display Member on multiple fields

            Is it possible to collect Nectar points via Trainline?